Prove $sum_{n = 1}^{p - 1} n^{p - 1} equiv (p - 1)! + p pmod {p^2}$ for $p$ being an odd prime












12












$begingroup$


I need to prove the following:




$$sum_{n = 1}^{p - 1} n^{p - 1} equiv (p - 1)! + p pmod {p^2}$$




...with $p$ being an odd prime number. The statement is obviously true for$pmod p$ because left-hand side is congruent to $-1 pmod p$ by Fermat's little theorem, and the right-hand side also turns out to be congruent to $-1 pmod p$ by Wilson's theorem. Now, I am not sure how to make a jump from$pmod p$ to$pmod {p^2}$, if that is even possible. Maybe the sum on the left could somehow be modified using the existence of the primitive root$pmod {p^2}$.



EDIT: Elementary solution can be found here: https://mathoverflow.net/a/319824/134054










share|cite|improve this question











$endgroup$












  • $begingroup$
    @MohammadZuhairKhan Yes, thanks. I have corrected the formatting
    $endgroup$
    – Oldboy
    Dec 29 '18 at 11:56










  • $begingroup$
    This is probably true for odd $p$
    $endgroup$
    – Maria Mazur
    Dec 29 '18 at 12:10










  • $begingroup$
    My bad, I have corrected the statement. I apologize for that.
    $endgroup$
    – Oldboy
    Dec 29 '18 at 12:13






  • 1




    $begingroup$
    @Oldboy Isn't Fermat's little theorem, only $a^p equiv a pmod p$ and not for the series?
    $endgroup$
    – toric_actions
    Dec 29 '18 at 12:46






  • 3




    $begingroup$
    Elementary proof can be found here: mathoverflow.net/a/319824/134054
    $endgroup$
    – Oldboy
    Dec 31 '18 at 17:29
















12












$begingroup$


I need to prove the following:




$$sum_{n = 1}^{p - 1} n^{p - 1} equiv (p - 1)! + p pmod {p^2}$$




...with $p$ being an odd prime number. The statement is obviously true for$pmod p$ because left-hand side is congruent to $-1 pmod p$ by Fermat's little theorem, and the right-hand side also turns out to be congruent to $-1 pmod p$ by Wilson's theorem. Now, I am not sure how to make a jump from$pmod p$ to$pmod {p^2}$, if that is even possible. Maybe the sum on the left could somehow be modified using the existence of the primitive root$pmod {p^2}$.



EDIT: Elementary solution can be found here: https://mathoverflow.net/a/319824/134054










share|cite|improve this question











$endgroup$












  • $begingroup$
    @MohammadZuhairKhan Yes, thanks. I have corrected the formatting
    $endgroup$
    – Oldboy
    Dec 29 '18 at 11:56










  • $begingroup$
    This is probably true for odd $p$
    $endgroup$
    – Maria Mazur
    Dec 29 '18 at 12:10










  • $begingroup$
    My bad, I have corrected the statement. I apologize for that.
    $endgroup$
    – Oldboy
    Dec 29 '18 at 12:13






  • 1




    $begingroup$
    @Oldboy Isn't Fermat's little theorem, only $a^p equiv a pmod p$ and not for the series?
    $endgroup$
    – toric_actions
    Dec 29 '18 at 12:46






  • 3




    $begingroup$
    Elementary proof can be found here: mathoverflow.net/a/319824/134054
    $endgroup$
    – Oldboy
    Dec 31 '18 at 17:29














12












12








12


8



$begingroup$


I need to prove the following:




$$sum_{n = 1}^{p - 1} n^{p - 1} equiv (p - 1)! + p pmod {p^2}$$




...with $p$ being an odd prime number. The statement is obviously true for$pmod p$ because left-hand side is congruent to $-1 pmod p$ by Fermat's little theorem, and the right-hand side also turns out to be congruent to $-1 pmod p$ by Wilson's theorem. Now, I am not sure how to make a jump from$pmod p$ to$pmod {p^2}$, if that is even possible. Maybe the sum on the left could somehow be modified using the existence of the primitive root$pmod {p^2}$.



EDIT: Elementary solution can be found here: https://mathoverflow.net/a/319824/134054










share|cite|improve this question











$endgroup$




I need to prove the following:




$$sum_{n = 1}^{p - 1} n^{p - 1} equiv (p - 1)! + p pmod {p^2}$$




...with $p$ being an odd prime number. The statement is obviously true for$pmod p$ because left-hand side is congruent to $-1 pmod p$ by Fermat's little theorem, and the right-hand side also turns out to be congruent to $-1 pmod p$ by Wilson's theorem. Now, I am not sure how to make a jump from$pmod p$ to$pmod {p^2}$, if that is even possible. Maybe the sum on the left could somehow be modified using the existence of the primitive root$pmod {p^2}$.



EDIT: Elementary solution can be found here: https://mathoverflow.net/a/319824/134054







elementary-number-theory modular-arithmetic






share|cite|improve this question















share|cite|improve this question













share|cite|improve this question




share|cite|improve this question








edited Jan 2 at 6:05







Oldboy

















asked Dec 29 '18 at 11:52









OldboyOldboy

9,47411138




9,47411138












  • $begingroup$
    @MohammadZuhairKhan Yes, thanks. I have corrected the formatting
    $endgroup$
    – Oldboy
    Dec 29 '18 at 11:56










  • $begingroup$
    This is probably true for odd $p$
    $endgroup$
    – Maria Mazur
    Dec 29 '18 at 12:10










  • $begingroup$
    My bad, I have corrected the statement. I apologize for that.
    $endgroup$
    – Oldboy
    Dec 29 '18 at 12:13






  • 1




    $begingroup$
    @Oldboy Isn't Fermat's little theorem, only $a^p equiv a pmod p$ and not for the series?
    $endgroup$
    – toric_actions
    Dec 29 '18 at 12:46






  • 3




    $begingroup$
    Elementary proof can be found here: mathoverflow.net/a/319824/134054
    $endgroup$
    – Oldboy
    Dec 31 '18 at 17:29


















  • $begingroup$
    @MohammadZuhairKhan Yes, thanks. I have corrected the formatting
    $endgroup$
    – Oldboy
    Dec 29 '18 at 11:56










  • $begingroup$
    This is probably true for odd $p$
    $endgroup$
    – Maria Mazur
    Dec 29 '18 at 12:10










  • $begingroup$
    My bad, I have corrected the statement. I apologize for that.
    $endgroup$
    – Oldboy
    Dec 29 '18 at 12:13






  • 1




    $begingroup$
    @Oldboy Isn't Fermat's little theorem, only $a^p equiv a pmod p$ and not for the series?
    $endgroup$
    – toric_actions
    Dec 29 '18 at 12:46






  • 3




    $begingroup$
    Elementary proof can be found here: mathoverflow.net/a/319824/134054
    $endgroup$
    – Oldboy
    Dec 31 '18 at 17:29
















$begingroup$
@MohammadZuhairKhan Yes, thanks. I have corrected the formatting
$endgroup$
– Oldboy
Dec 29 '18 at 11:56




$begingroup$
@MohammadZuhairKhan Yes, thanks. I have corrected the formatting
$endgroup$
– Oldboy
Dec 29 '18 at 11:56












$begingroup$
This is probably true for odd $p$
$endgroup$
– Maria Mazur
Dec 29 '18 at 12:10




$begingroup$
This is probably true for odd $p$
$endgroup$
– Maria Mazur
Dec 29 '18 at 12:10












$begingroup$
My bad, I have corrected the statement. I apologize for that.
$endgroup$
– Oldboy
Dec 29 '18 at 12:13




$begingroup$
My bad, I have corrected the statement. I apologize for that.
$endgroup$
– Oldboy
Dec 29 '18 at 12:13




1




1




$begingroup$
@Oldboy Isn't Fermat's little theorem, only $a^p equiv a pmod p$ and not for the series?
$endgroup$
– toric_actions
Dec 29 '18 at 12:46




$begingroup$
@Oldboy Isn't Fermat's little theorem, only $a^p equiv a pmod p$ and not for the series?
$endgroup$
– toric_actions
Dec 29 '18 at 12:46




3




3




$begingroup$
Elementary proof can be found here: mathoverflow.net/a/319824/134054
$endgroup$
– Oldboy
Dec 31 '18 at 17:29




$begingroup$
Elementary proof can be found here: mathoverflow.net/a/319824/134054
$endgroup$
– Oldboy
Dec 31 '18 at 17:29










1 Answer
1






active

oldest

votes


















1












$begingroup$

A Hint: notice that




$$sum^{p-1}_{n=1}n^{p-1}=frac{1}{p}sum^{p}_{n=1}C^{p-n}_{p}B_{p-n}p^n=sum^{p}_{n=1}C^{p-n}_{p}B_{p-n}p^{n-1}$$
where $B_k$ is the $k$-th Bernoulli number.




(For further information about Bernoulli number: https://www.bernoulli.org/)






share|cite|improve this answer











$endgroup$














    Your Answer








    StackExchange.ready(function() {
    var channelOptions = {
    tags: "".split(" "),
    id: "69"
    };
    initTagRenderer("".split(" "), "".split(" "), channelOptions);

    StackExchange.using("externalEditor", function() {
    // Have to fire editor after snippets, if snippets enabled
    if (StackExchange.settings.snippets.snippetsEnabled) {
    StackExchange.using("snippets", function() {
    createEditor();
    });
    }
    else {
    createEditor();
    }
    });

    function createEditor() {
    StackExchange.prepareEditor({
    heartbeatType: 'answer',
    autoActivateHeartbeat: false,
    convertImagesToLinks: true,
    noModals: true,
    showLowRepImageUploadWarning: true,
    reputationToPostImages: 10,
    bindNavPrevention: true,
    postfix: "",
    imageUploader: {
    brandingHtml: "Powered by u003ca class="icon-imgur-white" href="https://imgur.com/"u003eu003c/au003e",
    contentPolicyHtml: "User contributions licensed under u003ca href="https://creativecommons.org/licenses/by-sa/3.0/"u003ecc by-sa 3.0 with attribution requiredu003c/au003e u003ca href="https://stackoverflow.com/legal/content-policy"u003e(content policy)u003c/au003e",
    allowUrls: true
    },
    noCode: true, onDemand: true,
    discardSelector: ".discard-answer"
    ,immediatelyShowMarkdownHelp:true
    });


    }
    });














    draft saved

    draft discarded


















    StackExchange.ready(
    function () {
    StackExchange.openid.initPostLogin('.new-post-login', 'https%3a%2f%2fmath.stackexchange.com%2fquestions%2f3055767%2fprove-sum-n-1p-1-np-1-equiv-p-1-p-pmod-p2-for-p-b%23new-answer', 'question_page');
    }
    );

    Post as a guest















    Required, but never shown

























    1 Answer
    1






    active

    oldest

    votes








    1 Answer
    1






    active

    oldest

    votes









    active

    oldest

    votes






    active

    oldest

    votes









    1












    $begingroup$

    A Hint: notice that




    $$sum^{p-1}_{n=1}n^{p-1}=frac{1}{p}sum^{p}_{n=1}C^{p-n}_{p}B_{p-n}p^n=sum^{p}_{n=1}C^{p-n}_{p}B_{p-n}p^{n-1}$$
    where $B_k$ is the $k$-th Bernoulli number.




    (For further information about Bernoulli number: https://www.bernoulli.org/)






    share|cite|improve this answer











    $endgroup$


















      1












      $begingroup$

      A Hint: notice that




      $$sum^{p-1}_{n=1}n^{p-1}=frac{1}{p}sum^{p}_{n=1}C^{p-n}_{p}B_{p-n}p^n=sum^{p}_{n=1}C^{p-n}_{p}B_{p-n}p^{n-1}$$
      where $B_k$ is the $k$-th Bernoulli number.




      (For further information about Bernoulli number: https://www.bernoulli.org/)






      share|cite|improve this answer











      $endgroup$
















        1












        1








        1





        $begingroup$

        A Hint: notice that




        $$sum^{p-1}_{n=1}n^{p-1}=frac{1}{p}sum^{p}_{n=1}C^{p-n}_{p}B_{p-n}p^n=sum^{p}_{n=1}C^{p-n}_{p}B_{p-n}p^{n-1}$$
        where $B_k$ is the $k$-th Bernoulli number.




        (For further information about Bernoulli number: https://www.bernoulli.org/)






        share|cite|improve this answer











        $endgroup$



        A Hint: notice that




        $$sum^{p-1}_{n=1}n^{p-1}=frac{1}{p}sum^{p}_{n=1}C^{p-n}_{p}B_{p-n}p^n=sum^{p}_{n=1}C^{p-n}_{p}B_{p-n}p^{n-1}$$
        where $B_k$ is the $k$-th Bernoulli number.




        (For further information about Bernoulli number: https://www.bernoulli.org/)







        share|cite|improve this answer














        share|cite|improve this answer



        share|cite|improve this answer








        edited Dec 31 '18 at 10:37









        Martin Sleziak

        45k10123277




        45k10123277










        answered Dec 29 '18 at 13:42









        LauLau

        515315




        515315






























            draft saved

            draft discarded




















































            Thanks for contributing an answer to Mathematics Stack Exchange!


            • Please be sure to answer the question. Provide details and share your research!

            But avoid



            • Asking for help, clarification, or responding to other answers.

            • Making statements based on opinion; back them up with references or personal experience.


            Use MathJax to format equations. MathJax reference.


            To learn more, see our tips on writing great answers.




            draft saved


            draft discarded














            StackExchange.ready(
            function () {
            StackExchange.openid.initPostLogin('.new-post-login', 'https%3a%2f%2fmath.stackexchange.com%2fquestions%2f3055767%2fprove-sum-n-1p-1-np-1-equiv-p-1-p-pmod-p2-for-p-b%23new-answer', 'question_page');
            }
            );

            Post as a guest















            Required, but never shown





















































            Required, but never shown














            Required, but never shown












            Required, but never shown







            Required, but never shown

































            Required, but never shown














            Required, but never shown












            Required, but never shown







            Required, but never shown







            Popular posts from this blog

            Biblatex bibliography style without URLs when DOI exists (in Overleaf with Zotero bibliography)

            ComboBox Display Member on multiple fields

            Is it possible to collect Nectar points via Trainline?